Last visit was: 20 May 2024, 15:56 It is currently 20 May 2024, 15:56
Close
GMAT Club Daily Prep
Thank you for using the timer - this advanced tool can estimate your performance and suggest more practice questions. We have subscribed you to Daily Prep Questions via email.

Customized
for You

we will pick new questions that match your level based on your Timer History

Track
Your Progress

every week, we’ll send you an estimated GMAT score based on your performance

Practice
Pays

we will pick new questions that match your level based on your Timer History
Not interested in getting valuable practice questions and articles delivered to your email? No problem, unsubscribe here.
Close
Request Expert Reply
Confirm Cancel
SORT BY:
Date
Math Expert
Joined: 02 Sep 2009
Posts: 93351
Own Kudos [?]: 625341 [0]
Given Kudos: 81907
Send PM
Math Expert
Joined: 02 Sep 2009
Posts: 93351
Own Kudos [?]: 625341 [1]
Given Kudos: 81907
Send PM
Senior Manager
Senior Manager
Joined: 17 Jun 2022
Posts: 250
Own Kudos [?]: 125 [0]
Given Kudos: 67
Send PM
Math Expert
Joined: 02 Sep 2009
Posts: 93351
Own Kudos [?]: 625341 [1]
Given Kudos: 81907
Send PM
Re: M37-58 [#permalink]
1
Bookmarks
Expert Reply
Rabab36 wrote:
Bunuel wrote:
Official Solution:

If \((|x| - 2)(x + 5) < 0\), then which of the following must be true?

A. \(x > 2\)
B. \(x < 2\)
C. \(-2 < x < 2\)
D. \(-5 < x < 2\)
E. \(x < -5\)


\((|x| - 2)(x + 5) < 0\) means that \(|x| - 2\) and \(x + 5\) must have the opposite signs.

CASE 1: \(|x| - 2 > 0\) and \(x + 5 < 0\):


\(|x| - 2 > 0\) means that \(x < -2\) or \(x > 2\);

\(x + 5 < 0\) means that \(x < -5\).

Intersection of these ranges is \(x< -5\).

CASE 2: \(|x| - 2 < 0\) and \(x + 5 > 0\):

\(|x| - 2 < 0\) means that \(-2 < x < 2\);

\(x + 5 > 0\) means that \(x > -5\).

Intersection of these ranges is \(-2 < x < 2\).

So, we have that \((|x| - 2)(x + 5) < 0\) means that \(x < -5\) or \(-2 < x < 2\). ANY \(x\) from these possible ranges will for sure be less than 2 (option B).

To explaining other options:

\(x > 2\) (A) is not true because \(x\) could say be 0.

\(-2 < x < 2\) (C) is not true because \(x\) could say be -10.

\(-5 < x < 2\) (D) is not true because \(x\) could say be -10.

\(x < -5\) (E) is not true because \(x\) could say be 0.


Answer: B


Bunuel but for x=-4 and -3 the inequality do not hold true if we take x<2 as an option


The question essentially boils down to: if x < -5 or -2 < x < 2, which of the following statements must be true?

Option B, which says x < 2, must be true since whatever x is, and remember it can only be less than -5 or between -2 and 2, it must be less than 2. Check it yourself:

<------------(-5)----(-2)------(2)------------>

x is located within the green zone, and any x from that region will undoubtedly be less than 2.

This question belongs to one of the most challenging subgroups of Must/Could be true questions, and many people get them wrong. To better understand the underlying concept, practice other Trickiest Inequality Questions Type: Confusing Ranges.

I hope this helps.
Manager
Manager
Joined: 30 Apr 2023
Status:A man of Focus, Commitment and Shear Will
Posts: 136
Own Kudos [?]: 58 [0]
Given Kudos: 72
Location: India
GMAT 1: 710 Q50 V36
WE:Investment Banking (Investment Banking)
Send PM
Re: M37-58 [#permalink]
Bunuel wrote:
If \((|x| - 2)(x + 5) < 0\), then which of the following must be true?

A. \(x > 2\)
B. \(x < 2\)
C. \(-2 < x < 2\)
D. \(-5 < x < 2\)
E. \(x < -5\)


This is a quality question IMO. Which is why it had me scratching my head for 2.5mins after solving it in 2.5mins before marking and moving on! I had arrived at both ranges but moved on to not waste any more time. Ahhh the frustration.

Benuel - can you advise me on how to decide between plug and play and solve the inequality for range? In which case is one better than the other?
Intern
Intern
Joined: 03 May 2020
Posts: 5
Own Kudos [?]: 1 [0]
Given Kudos: 29
Location: Germany
Schools: AGSM
GPA: 3.3
Send PM
M37-58 [#permalink]
Hello Bunuel,
Thankyou for the official explanation and in general for the wonderful support you provide.

I feel that many (including myself) misinterpret this Question - specially since its not clear which the superset is, the OPTION or the Question Stem.

The way I understood it, the question names a range for 'x', and then asks, which of the following options is COMPLETELY ENGULFED by the aforementioned range.
"If XYZ is true, then what MUST BE TRUE?"
We are asked, what FOLLOWS CONSEQUENTIALLY if the Inequality in the Question Stem is true.

In other words, what fits completely in the Superset below?
<------------(-5)----(-2)------(2)------------>

The way you answer it, the question should be -
"Which OPTION MUST BE TRUE, so that this INEQUALITY COULD BE valid?"

What do you think?
Math Expert
Joined: 02 Sep 2009
Posts: 93351
Own Kudos [?]: 625341 [1]
Given Kudos: 81907
Send PM
Re: M37-58 [#permalink]
1
Kudos
Expert Reply
VibhasJ wrote:
Hello Bunuel,
Thankyou for the official explanation and in general for the wonderful support you provide.

I feel that many (including myself) misinterpret this Question - specially since its not clear which the superset is, the OPTION or the Question Stem.

The way I understood it, the question names a range for 'x', and then asks, which of the following options is COMPLETELY ENGULFED by the aforementioned range.
"If XYZ is true, then what MUST BE TRUE?"
We are asked, what FOLLOWS CONSEQUENTIALLY if the Inequality in the Question Stem is true.

In other words, what fits completely in the Superset below?
<------------(-5)----(-2)------(2)------------>

The way you answer it, the question should be -
"Which OPTION MUST BE TRUE, so that this INEQUALITY COULD BE valid?"

What do you think?


Firstly, I understand this question is a tricky one - it's among the more challenging types and it can take some time to fully understand. Don't worry, you're not alone!

The wording and solution of the question are spot on, I assure you. To further help, I've expanded on the solution in this post. If you're still finding it tough to understand, I strongly recommend practicing with similar logic-based questions from this helpful collection: Trickiest Inequality Questions Type: Confusing Ranges.

Hope this clears things up for you!
Manager
Manager
Joined: 23 Jan 2024
Posts: 91
Own Kudos [?]: 17 [0]
Given Kudos: 110
Send PM
Re: M37-58 [#permalink]
Answer process is right but choice of answer is wrong? 

Following your explanations for rejection of "wrong answers", same can be said for B. Since x is either in x<-5 or -2<x<2 

Answer B is also WRONG. X<2 includes numbers such as -3 and -4 which are not part of the valid range...
Bunuel wrote:
Official Solution:

If \((|x| - 2)(x + 5) < 0\), then which of the following must be true?

A. \(x > 2\)
B. \(x < 2\)
C. \(-2 < x < 2\)
D. \(-5 < x < 2\)
E. \(x < -5\)


\((|x| - 2)(x + 5) < 0\) means that \(|x| - 2\) and \(x + 5\) must have the opposite signs.

CASE 1: \(|x| - 2 > 0\) and \(x + 5 < 0\):

\(|x| - 2 > 0\) means that \(x < -2\) or \(x > 2\);

\(x + 5 < 0\) means that \(x < -5\).

Intersection of these ranges is \(x< -5\).

CASE 2: \(|x| - 2 < 0\) and \(x + 5 > 0\):

\(|x| - 2 < 0\) means that \(-2 < x < 2\);

\(x + 5 > 0\) means that \(x > -5\).

Intersection of these ranges is \(-2 < x < 2\).

So, we have that \((|x| - 2)(x + 5) < 0\) means that \(x < -5\) or \(-2 < x < 2\). ANY \(x\) from these possible ranges will for sure be less than 2 (option B).

To explaining other options:

\(x > 2\) (A) is not true because \(x\) could say be 0.

\(-2 < x < 2\) (C) is not true because \(x\) could say be -10.

\(-5 < x < 2\) (D) is not true because \(x\) could say be -10.

\(x < -5\) (E) is not true because \(x\) could say be 0.


Answer: B

­
Math Expert
Joined: 02 Sep 2009
Posts: 93351
Own Kudos [?]: 625341 [0]
Given Kudos: 81907
Send PM
Re: M37-58 [#permalink]
Expert Reply
unicornilove wrote:
Answer process is right but choice of answer is wrong? 

Following your explanations for rejection of "wrong answers", same can be said for B. Since x is either in x<-5 or -2<x<2 

Answer B is also WRONG. X<2 includes numbers such as -3 and -4 which are not part of the valid range...
Bunuel wrote:
Official Solution:

If \((|x| - 2)(x + 5) < 0\), then which of the following must be true?

A. \(x > 2\)
B. \(x < 2\)
C. \(-2 < x < 2\)
D. \(-5 < x < 2\)
E. \(x < -5\)


\((|x| - 2)(x + 5) < 0\) means that \(|x| - 2\) and \(x + 5\) must have the opposite signs.

CASE 1: \(|x| - 2 > 0\) and \(x + 5 < 0\):

\(|x| - 2 > 0\) means that \(x < -2\) or \(x > 2\);

\(x + 5 < 0\) means that \(x < -5\).

Intersection of these ranges is \(x< -5\).

CASE 2: \(|x| - 2 < 0\) and \(x + 5 > 0\):

\(|x| - 2 < 0\) means that \(-2 < x < 2\);

\(x + 5 > 0\) means that \(x > -5\).

Intersection of these ranges is \(-2 < x < 2\).

So, we have that \((|x| - 2)(x + 5) < 0\) means that \(x < -5\) or \(-2 < x < 2\). ANY \(x\) from these possible ranges will for sure be less than 2 (option B).

To explaining other options:

\(x > 2\) (A) is not true because \(x\) could say be 0.

\(-2 < x < 2\) (C) is not true because \(x\) could say be -10.

\(-5 < x < 2\) (D) is not true because \(x\) could say be -10.

\(x < -5\) (E) is not true because \(x\) could say be 0.


Answer: B­

 

Please review the discussion above more carefully and follow the links to the similar questions. We know that \(x < -5\) or \(-2 < x < 2\). Given these conditions, the statement \(x < 2\) holds true for ANY x within the ranges of \(x < -5\) or \(-2 < x < 2\). So, if you pick ANY x from the ranges \(x < -5\) or \(-2 < x < 2\), it would be correct to say that this value of x is less than 2.


 
GMAT Club Bot
Re: M37-58 [#permalink]
Moderator:
Math Expert
93351 posts